Stay ahead of learning milestones! Enroll in a class over the summer!

G
Topic
First Poster
Last Poster
Nepal TST DAY 1 Problem 1
Bata325   6
N an hour ago by Mathdreams
Source: Nepal TST 2025 p1
Consider a triangle $\triangle ABC$ and some point $X$ on $BC$. The perpendicular from $X$ to $AB$ intersects the circumcircle of $\triangle AXC$ at $P$ and the perpendicular from $X$ to $AC$ intersects the circumcircle of $\triangle AXB$ at $Q$. Show that the line $PQ$ does not depend on the choice of $X$.(Shining Sun, USA)
6 replies
Bata325
Yesterday at 1:21 PM
Mathdreams
an hour ago
NEPAL TST DAY 2 PROBLEM 2
Tony_stark0094   4
N an hour ago by Mathdreams
Kritesh manages traffic on a $45 \times 45$ grid consisting of 2025 unit squares. Within each unit square is a car, facing either up, down, left, or right. If the square in front of a car in the direction it is facing is empty, it can choose to move forward. Each car wishes to exit the $45 \times 45$ grid.

Kritesh realizes that it may not always be possible for all the cars to leave the grid. Therefore, before the process begins, he will remove $k$ cars from the $45 \times 45$ grid in such a way that it becomes possible for all the remaining cars to eventually exit the grid.

What is the minimum value of $k$ that guarantees that Kritesh's job is possible?
4 replies
Tony_stark0094
Today at 8:37 AM
Mathdreams
an hour ago
Abelkonkurransen 2025 1a
Lil_flip38   1
N 2 hours ago by MathLuis
Source: abelkonkurransen
Peer and Solveig are playing a game with $n$ coins, all of which show $M$ on one side and $K$ on the opposite side. The coins are laid out in a row on the table. Peer and Solveig alternate taking turns. On his turn, Peer may turn over one or more coins, so long as he does not turn over two adjacent coins. On her turn, Solveig picks precisely two adjacent coins and turns them over. When the game begins, all the coins are showing $M$. Peer plays first, and he wins if all the coins show $K$ simultaneously at any time. Find all $n\geqslant 2$ for which Solveig can keep Peer from winning.
1 reply
Lil_flip38
Mar 20, 2025
MathLuis
2 hours ago
cos k theta and cos(k + 1) theta are both rational
N.T.TUAN   12
N 2 hours ago by Ilikeminecraft
Source: USA Team Selection Test 2007
Let $ \theta$ be an angle in the interval $ (0,\pi/2)$. Given that $ \cos \theta$ is irrational, and that $ \cos k \theta$ and $ \cos[(k + 1)\theta ]$ are both rational for some positive integer $ k$, show that $ \theta = \pi/6$.
12 replies
N.T.TUAN
Dec 8, 2007
Ilikeminecraft
2 hours ago
Problem 3 IMO 2005 (Day 1)
Valentin Vornicu   119
N 2 hours ago by MTA_2024
Let $x,y,z$ be three positive reals such that $xyz\geq 1$. Prove that
\[ \frac { x^5-x^2 }{x^5+y^2+z^2} + \frac {y^5-y^2}{x^2+y^5+z^2} + \frac {z^5-z^2}{x^2+y^2+z^5} \geq 0 . \]
Hojoo Lee, Korea
119 replies
Valentin Vornicu
Jul 13, 2005
MTA_2024
2 hours ago
Rhombus EVAN
62861   71
N 2 hours ago by ihategeo_1969
Source: USA January TST for IMO 2017, Problem 2
Let $ABC$ be a triangle with altitude $\overline{AE}$. The $A$-excircle touches $\overline{BC}$ at $D$, and intersects the circumcircle at two points $F$ and $G$. Prove that one can select points $V$ and $N$ on lines $DG$ and $DF$ such that quadrilateral $EVAN$ is a rhombus.

Danielle Wang and Evan Chen
71 replies
62861
Feb 23, 2017
ihategeo_1969
2 hours ago
A and B play a game
EthanWYX2009   3
N 2 hours ago by nitr4m
Source: 2025 TST 23
Let \( n \geq 2 \) be an integer. Two players, Alice and Bob, play the following game on the complete graph \( K_n \): They take turns to perform operations, where each operation consists of coloring one or two edges that have not been colored yet. The game terminates if at any point there exists a triangle whose three edges are all colored.

Prove that there exists a positive number \(\varepsilon\), Alice has a strategy such that, no matter how Bob colors the edges, the game terminates with the number of colored edges not exceeding
\[
\left( \frac{1}{4} - \varepsilon \right) n^2 + n.
\]
3 replies
EthanWYX2009
Mar 29, 2025
nitr4m
2 hours ago
Problem 3
SlovEcience   1
N 2 hours ago by kokcio
Find all real numbers \( k \) such that the following inequality holds for all \( a, b, c \geq 0 \):

\[
ab + bc + ca \leq \frac{(a + b + c)^2}{3} + k \cdot \max \{ (a - b)^2, (b - c)^2, (c - a)^2 \} \leq a^2 + b^2 + c^2
\]
1 reply
SlovEcience
Apr 9, 2025
kokcio
2 hours ago
Inequality with a,b,c
GeoMorocco   8
N 2 hours ago by GeoMorocco
Source: Morocco Training 2025
Let $   a,b,c   $ be positive real numbers such that : $   ab+bc+ca=3   $ . Prove that : $$\frac{a\sqrt{3+bc}}{b+c}+\frac{b\sqrt{3+ca}}{c+a}+\frac{c\sqrt{3+ab}}{a+b}\ge a+b+c $$
8 replies
GeoMorocco
Apr 10, 2025
GeoMorocco
2 hours ago
prove that any quadrilateral satisfying this inequality is a trapezoid
mqoi_KOLA   1
N 2 hours ago by vgtcross
Prove that any quadrilateral satisfying this inequality is a Trapezoid/trapzium $$
|r - p| < q + s < r + p
$$where $p,r$ are lengths of parallel sides and $q,s$ are other two sides.
1 reply
mqoi_KOLA
Today at 3:48 AM
vgtcross
2 hours ago
Prove that there exists a convex 1990-gon
orl   13
N 3 hours ago by akliu
Source: IMO 1990, Day 2, Problem 6, IMO ShortList 1990, Problem 16 (NET 1)
Prove that there exists a convex 1990-gon with the following two properties :

a.) All angles are equal.
b.) The lengths of the 1990 sides are the numbers $ 1^2$, $ 2^2$, $ 3^2$, $ \cdots$, $ 1990^2$ in some order.
13 replies
orl
Nov 11, 2005
akliu
3 hours ago
Cosine Law and Leibniz's Theorem
narutomath   3
N Jul 9, 2016 by Virgil Nicula
The hypotenuse of a right triangle has length 1.The centroid of the triangle lies on the inscribed circle. Find the perimeter of the triangle.

I know a solution with Leibniz's Theorem, but the problem is in Cosine Law section. Can someone offer me a solution without Leibniz?
3 replies
narutomath
Jul 7, 2016
Virgil Nicula
Jul 9, 2016
Cosine Law and Leibniz's Theorem
G H J
G H BBookmark kLocked kLocked NReply
The post below has been deleted. Click to close.
This post has been deleted. Click here to see post.
narutomath
101 posts
#1 • 1 Y
Y by Adventure10
The hypotenuse of a right triangle has length 1.The centroid of the triangle lies on the inscribed circle. Find the perimeter of the triangle.

I know a solution with Leibniz's Theorem, but the problem is in Cosine Law section. Can someone offer me a solution without Leibniz?
Z K Y
The post below has been deleted. Click to close.
This post has been deleted. Click here to see post.
suli
1498 posts
#2 • 2 Y
Y by Adventure10, Mango247
I will offer my solution if you can tell me what is Leibniz's theorem.

Solution
This post has been edited 1 time. Last edited by suli, Jul 7, 2016, 4:33 PM
Z K Y
The post below has been deleted. Click to close.
This post has been deleted. Click here to see post.
george_54
1585 posts
#3 • 1 Y
Y by Adventure10
suli wrote:
I will offer my solution if you can tell me what is Leibniz's theorem.

If $G$ is the centroid of a triangle $ABC$ and $M$ is any point of the plane, then:

$\boxed{M{A^2} + M{B^2} + M{C^2} = 3M{G^2} + \frac{1}{3}({a^2} + {b^2} + {c^2})}$
Z K Y
The post below has been deleted. Click to close.
This post has been deleted. Click here to see post.
Virgil Nicula
7054 posts
#4 • 2 Y
Y by Adventure10, Mango247
See PP12 from here and PP19 from here.
This post has been edited 1 time. Last edited by Virgil Nicula, Jul 9, 2016, 4:38 PM
Z K Y
N Quick Reply
G
H
=
a